PT57.S2.Q25 - The law of the city of Weston regarding contributions

paulmv.benthempaulmv.benthem Alum Member
edited June 2018 in Logical Reasoning 1032 karma

Hey everyone,

So, I'm working through my analysis of this question since, yes, I got it wrong on my last section drill, and I'd be interested to hear some of all you smart people's thoughts on the question/choices. For context, it's a MBT. As I read it, the contribution law outlined in the stimulus could be translated logically as,

Contribution of $100 by a nonresident who was not a former resident ---> contribution must be registered with the city council

The author notes that no nonresidents contributed to a particular candidate's campaign. So, from my understanding, the sufficient condition outlined in the stimulus is failed (but not necessarily the necessary condition). So, when I glanced at "C," I immediately thought that they were trying to tempt me with a mistaken negation, purporting that it must be true that no contributions to the candidate's campaign needed to be registered with the council. The thought that popped into my mind was, "Well, but there could be other reasons why a contribution might need to be registered with the city council." But, in this case it seems that I and the LSAT writers are not of one mind. :P

Is part of my misunderstanding rooted in that it introduces the particular law in question as "The law..." rather than "A law..."? Should that indicate to me that there is only one law and, thus, only one reason for registering a contribution with the city council?

Love to hear you're thoughts!

Admin note: edited title
https://7sage.com/lsat_explanations/lsat-57-section-2-question-25/

Comments

  • FixedDiceFixedDice Member
    1804 karma

    Is part of my misunderstanding rooted in that it introduces the particular law in question as "The law..." rather than "A law..."? Should that indicate to me that there is only one law and, thus, only one reason for registering a contribution with the city council?

    Spot on. There's only one law, so the conditional statement is really a biconditional. Therefore, if the sufficient condition fails, so does the necessary condition.

Sign In or Register to comment.